Những câu hỏi liên quan
Lê Chí Cường
Xem chi tiết
Nguyễn Trí dũng
4 tháng 12 2016 lúc 22:14

Theo bài ra ta có \(0\le a\le b\le c\) nên b\(+\)\(\ge\)2b

Do đó suy ra \(\frac{2a^2}{b+c}\le\frac{2a^2}{2b}\)suy ra \(\frac{2a^2}{b+c}\le\frac{a^2}{b}\)

Chưng minh tương tự có \(\frac{2b^2}{c+a}\le\frac{b^2}{c}\)và \(\frac{2c^2}{a+b}\le\frac{c^2}{a}\)

Cộng vế với vế của các bđt cùng chiều trên ta sẽ suy ra điều phải chứng minh

#nga

alibaba nguyễn
4 tháng 12 2016 lúc 22:30

Sai rồi nếu như theo cách chứng minh của bạn thì ta có: a + c \(\ge2c\)cái này vô lý. 

Minh Anh
5 tháng 12 2016 lúc 12:11

Làm bừa, sai sót sửa giùm mình nhé :| 

bach nhac lam
Xem chi tiết
Ngô Bá Hùng
18 tháng 11 2019 lúc 20:54

1. Vai trò a, b, c như nhau. Không mất tính tổng quát. Giả sử \(a\ge b\ge0\)

\(ab+bc+ca=3\). Do đó \(ab\ge1\)

Ta cần chứng minh rằng \(\frac{1}{1+a^2}+\frac{1}{1+b^2}\ge\frac{2}{1+ab}\left(1\right)\)

\(\frac{2}{1+ab}+\frac{1}{1+c^2}\ge\frac{3}{2}\left(2\right)\)

Thật vậy: \(\left(1\right)\Leftrightarrow\frac{1}{1+a^2}-\frac{1}{1+ab}+\frac{1}{1+b^2}-\frac{1}{1+ab}\ge0\\ \Leftrightarrow\left(ab-a^2\right)\left(1+b^2\right)+\left(ab-b^2\right)\left(1+a^2\right)\ge0\\ \Leftrightarrow\left(a-b\right)\left[-a\left(1+b^2\right)+b\left(1+a^2\right)\right]\ge0\\ \Leftrightarrow\left(a-b\right)^2\left(ab-1\right)\ge0\left(BĐT:đúng\right)\)

\(\left(2\right)\Leftrightarrow c^2+3-ab\ge3abc^2\\ \Leftrightarrow c^2+ca+bc\ge3abc^2\Leftrightarrow a+b+c\ge3abc\)

BĐT đúng, vì \(\left(a+b+c\right)^2>3\left(ab+bc+ca\right)=q\)

\(ab+bc+ca\ge3\sqrt[3]{\left(abc\right)^2}\)

Nên \(a+b+c\ge3\ge3abc\)

Từ (1) và (2) ta có \(\frac{1}{1+a^2}+\frac{1}{1+b^2}+\frac{1}{1+c^2}\ge\frac{3}{2}\)

Dấu ''='' xảy ra \(\Leftrightarrow a=b=c=1\)

Khách vãng lai đã xóa
Ngô Bá Hùng
18 tháng 11 2019 lúc 21:16

Áp dụng BĐT Cauchy dạng \(\frac{9}{x+y+z}\le\frac{1}{x}+\frac{1}{y}+\frac{1}{z}\), ta được

\(\frac{9}{a+3b+2c}=\frac{1}{a+c+b+c+2b}\le\frac{1}{9}\left(\frac{1}{a+c}+\frac{1}{b+c}+\frac{1}{2b}\right)\)

Do đó ta được

\(\frac{ab}{a+3b+2c}\le\frac{ab}{9}\left(\frac{1}{a+c}+\frac{1}{b+c}+\frac{1}{2b}\right)=\frac{1}{9}\left(\frac{ab}{a+c}+\frac{ab}{b+c}+\frac{a}{2}\right)\)

Hoàn toàn tương tự ta được

\(\frac{bc}{2a+b+3c}\le\frac{1}{9}\left(\frac{bc}{a+b}+\frac{bc}{b+c}+\frac{b}{2}\right);\frac{ac}{3a+2b+c}\le\frac{1}{9}\left(\frac{ac}{a+b}+\frac{ac}{b+c}+\frac{c}{2}\right)\)

Cộng theo vế các BĐT trên ta được

\(\frac{ab}{a+3b+2c}+\frac{bc}{b+3c+2a}+\frac{ca}{c+3a+2b}\le\frac{1}{9}\left(\frac{ac+bc}{a+b}+\frac{ab+ac}{b+c}+\frac{bc+ab}{a+c}+\frac{a+b+c}{2}\right)=\frac{a+b+c}{6}\)Vậy BĐT đc CM

ĐẲng thức xảy ra khi và chỉ khi a = b = c >0

Khách vãng lai đã xóa
Akai Haruma
18 tháng 11 2019 lúc 22:38

Bài 2:

Áp dụng BĐT AM-GM:

\(a^2+2b^2+c^2=(a^2+b^2)+(a^2+c^2)\geq 2\sqrt{(a^2+b^2)(a^2+c^2)}\geq 2\sqrt{\frac{(a+b)^2}{2}.\frac{(a+c)^2}{2}}=(a+b)(a+c)\)

\(\Rightarrow \frac{ab^2}{a^2+2b^2+c^2}\leq \frac{ab^2}{(a+b)(a+c)}\)

Hoàn toàn tương tự với các phân thức còn lại:

\(\Rightarrow \text{VT}\leq \sum \frac{ab^2}{(a+b)(a+c)}=\frac{a^2b^2+b^2c^2+c^2a^2+abc(a+b+c)}{(a+b)(b+c)(c+a)}\)

Ta cần CM: \(\frac{a^2b^2+b^2c^2+c^2a^2+abc(a+b+c)}{(a+b)(b+c)(c+a)}\leq \frac{a+b+c}{4}\)

\(\Leftrightarrow 4(a^2b^2+b^2c^2+c^2a^2)+4abc(a+b+c)\leq (a+b+c)(a+b)(b+c)(c+a)\)

\(\Leftrightarrow 4(a^2b^2+b^2c^2+c^2a^2)+4abc(a+b+c)\leq (a+b+c)(a+b)(b+c)(c+a)\)

\(\Leftrightarrow 4(a^2b^2+b^2c^2+c^2a^2)+4abc(a+b+c)\leq (a+b+c)[(a+b+c)(ab+bc+ac)-abc]\)

\(\Leftrightarrow 2(a^2b^2+b^2c^2+c^2a^2)\leq (a^3b+ab^3)+(bc^3+b^3c)+(ca^3+c^3a)\)

(dễ thấy luôn đúng do theo BĐT AM-GM)

Do đó ta có đpcm.

Dấu "=" xảy ra khi $a=b=c$

Khách vãng lai đã xóa
Nguyễn Ngọc Khánh Ly
Xem chi tiết
Kudo Shinichi
Xem chi tiết
Kudo Shinichi
6 tháng 7 2016 lúc 21:00

Trả lời hộ mình đi

Hùng Quân Mai
Xem chi tiết
Duong Nguyen Tuan
Xem chi tiết
Kiệt Nguyễn
30 tháng 5 2020 lúc 20:23

Ta có: \(a^2+2b+3=\left(a^2+1\right)+2\left(b+1\right)\ge2\left(a+b+1\right)\)

Tương tự ta có: \(b^2+2c+3\ge2\left(b+c+1\right)\)\(c^2+2a+3\ge2\left(c+a+1\right)\)

Từ đó suy ra\(\frac{a}{a^2+2b+3}+\frac{b}{b^2+2c+3}+\frac{c}{c^2+2a+3}\)\(\le\frac{a}{2\left(a+b+1\right)}+\frac{b}{2\left(b+c+1\right)}+\frac{c}{2\left(c+a+1\right)}\)

\(=\frac{1}{2}\left(\frac{a}{a+b+1}+\frac{b}{b+c+1}+\frac{c}{c+a+1}\right)\)

Đặt \(K=\frac{a}{a+b+1}+\frac{b}{b+c+1}+\frac{c}{c+a+1}\), ta đi chứng minh \(K\le1\)

Thật vậy: \(3-K=\frac{b+1}{a+b+1}+\frac{c+1}{b+c+1}+\frac{a+1}{c+a+1}\)

\(=\frac{\left(b+1\right)^2}{\left(b+1\right)\left(a+b+1\right)}+\frac{\left(c+1\right)^2}{\left(c+1\right)\left(b+c+1\right)}+\frac{\left(a+1\right)^2}{\left(a+1\right)\left(c+a+1\right)}\)

\(\ge\frac{\left(a+b+c+3\right)^2}{\left(b+1\right)\left(a+b+1\right)+\left(c+1\right)\left(b+c+1\right)+\left(a+1\right)\left(c+a+1\right)}\)(*)

Ta có: \(\left(b+1\right)\left(a+b+1\right)+\left(c+1\right)\left(b+c+1\right)+\left(a+1\right)\left(c+a+1\right)\)\(=3\left(a+b+c\right)+ab+bc+ca+a^2+b^2+c^2+3\)

(Mình gõ bằng chương trình Universal Math Solver, không hiện ảnh thì vô thống kê hỏi đáp của mình, ngày 30/5/2020 vào lúc 8:25)

\(=\frac{1}{2}\left[\left(a+b+c\right)^2+6\left(a+b+c\right)+9\right]=\frac{1}{2}\left(a+b+c+3\right)^2\)(**)

Từ (*) và (**) suy ra \(3-K\ge\frac{\left(a+b+c+3\right)^2}{\frac{1}{2}\left(a+b+c+3\right)^2}=2\Rightarrow K\le1\)

Vậy ta có điều phải chứng minh

Đẳng thức xảy ra khi a = b = c = 1

Khách vãng lai đã xóa
Thanh Tùng DZ
30 tháng 5 2020 lúc 20:31

Áp dụng BĐT Cô-si,ta có :

\(a^2+1\ge2a\)

\(\Rightarrow\frac{a}{a^2+2b+3}\le\frac{a}{2a+2b+2}=\frac{1}{2}\left(\frac{a}{a+b+1}\right)\)

Tương tự : \(\frac{b}{b^2+2c+3}\le\frac{1}{2}\left(\frac{b}{b+c+1}\right);\frac{c}{c^2+2a+3}\le\frac{1}{2}\left(\frac{c}{c+a+1}\right)\)

\(\Rightarrow\frac{a}{a^2+2b+3}+\frac{b}{b^2+2c+3}+\frac{c}{c^2+2a+3}\le\frac{1}{2}\left(\frac{a}{a+b+1}+\frac{b}{b+c+1}+\frac{c}{c+a+1}\right)\)

Áp dụng BĐT Bu-nhi-a-cốp-ski,ta có :

\(\frac{a}{a+b+1}=\frac{a\left(a+b+c^2\right)}{\left(a+b+1\right)\left(a+b+c^2\right)}\le\frac{a^2+ab+ac^2}{\left(a^2+b^2+c^2\right)^2}=\frac{a^2+ab+ac^2}{9}\)

TT : ...

Cộng lại ta được :

\(\frac{a}{a+b+1}+\frac{b}{b+c+1}+\frac{c}{c+a+1}\le\frac{a^2+ab+ac^2}{9}+\frac{b^2+bc+ba^2}{9}+\frac{c^2+ca+cb^2}{9}\)

\(=\frac{a^2+b^2+c^2+ab+bc+ac+ac^2+ba^2+cb^2}{9}\le\frac{3+3+3}{9}=1\)

\(\Rightarrow\frac{a}{a^2+2b+3}+\frac{b}{b^2+2c+3}+\frac{c}{c^2+2a+3}\le\frac{1}{2}\)

Dấu "=" xảy ra khi a = b = c = 1

Khách vãng lai đã xóa
Trang Nana
Xem chi tiết
Nguyễn Việt Lâm
13 tháng 6 2020 lúc 18:05

Đặt vế trái là P

\(P=\frac{1}{a^2+b^2+b^2+1+2}+\frac{1}{b^2+c^2+c^2+1+2}+\frac{1}{c^2+a^2+a^2+1+2}\)

\(P\le\frac{1}{2ab+2b+2}+\frac{1}{2bc+2c+2}+\frac{1}{2ca+2a+2}=\frac{1}{2}\left(\frac{1}{ab+b+1}+\frac{1}{bc+c+1}+\frac{1}{ca+a+1}\right)\)

\(P\le\frac{1}{2}\left(\frac{1}{ab+b+1}+\frac{abc}{bc+c+abc}+\frac{b}{abc+ab+b}\right)\)

\(P\le\frac{1}{2}\left(\frac{1}{ab+b+1}+\frac{ab}{b+1+ab}+\frac{b}{1+ab+b}\right)=\frac{1}{2}\)

Dấu "=" xảy ra khi \(a=b=c=1\)

Không Có Tên
Xem chi tiết
pham trung thanh
22 tháng 4 2018 lúc 16:51

Ngược dấu rồi

Không Có Tên
22 tháng 4 2018 lúc 16:53

Mk sửa r đó. H giúp mk vs. Cảm ơn

pham trung thanh
22 tháng 4 2018 lúc 16:55

Cô-si mẫu suy ra: 

\(A\le\frac{1}{2}\left(\frac{1}{ab+a+1}+\frac{1}{bc+b+1}+\frac{1}{ca+c+1}\right)\)

Dễ cm biểu thức trong ngoặc = 1. 

Suy ra A <=1/2

Dấu = khi a=b=c=1

Nghiêm Thị Nhân Đức
Xem chi tiết
Copxki Minh
2 tháng 12 2020 lúc 22:25

Đặt \(\left(\frac{1}{a},\frac{1}{b},\frac{1}{c}\right)=\left(x,y,z\right)\)

\(x+y+z\ge\frac{x^2+2xy}{2x+y}+\frac{y^2+2yz}{2y+z}+\frac{z^2+2zx}{2z+x}\)

\(\Leftrightarrow x+y+z\ge\frac{3xy}{2x+y}+\frac{3yz}{2y+z}+\frac{3zx}{2z+x}\)

\(\frac{3xy}{2x+y}\le\frac{3}{9}xy\left(\frac{1}{x}+\frac{1}{x}+\frac{1}{y}\right)=\frac{1}{3}\left(x+2y\right)\)

\(\Rightarrow\Sigma_{cyc}\frac{3xy}{2x+y}\le\frac{1}{3}\left[\left(x+2y\right)+\left(y+2z\right)+\left(z+2x\right)\right]=x+y+z\)

Dấu "=" xảy ra khi x=y=z

Khách vãng lai đã xóa